13

Consider the following problem $$ \max _{p_i} \sum_{i=1}^n p_i^{(1-p_i)^a} $$ s.t. $p_i \in [0,1]$ and $\sum_{i=1}^n p_i= 1$ for some given $a>0$. In other words, ${\bf p} = \{ p_i\}$ is a probability vector.

Question: Can we find ${\bf p}$ that maximizes the problem? If not, could we find a 'good' upper bound?

For the case of $n=2$, we have to maximize \begin{align} p^{(1-p)^a} + (1-p)^{p^a}, \end{align} which, from simulations, occurs at $p=1/2$ for all $a>0$.

Edit: Originally, I thought that the maximum occurs by choosing $p_i=1/n$. However, this appears not to be the case as given by the example in the chat for $n=3$.

Boby
  • 6,381
  • 4
    I don't know the solution, but it's not true that the maximum is at $\mathbf{1}/n$ always. Numerically, for $a = 0.2,$ $2 \cdot (1/2)^{1/2^{0.2}} = 1.09... > 1.08... = 3 \cdot (1/3)^{(2/3)^{0.2}}$, which means that for $n = 3, a = 0.2,$ the vector $(1/2,1/2,0)$ has a larger value than $(1/3,1/3,1/3)$ – stochasticboy321 Nov 15 '24 at 14:29
  • @stochasticboy321 Thanks. Let me update the question based on your response – Boby Nov 15 '24 at 14:32
  • 3
    Try your luck with EV theorem - it can surprisingly kill problems that attains optimal values at almost all equal or just two equal and everything else is zero. – dezdichado Nov 16 '24 at 20:27
  • For instance, if one can establish the inequality: $$2\left(\frac{x+y}{2}\right)^{(1-\frac{x+y}{2})^a}\leq x^{(1-x)^a} + y^{(1-y)^a},$$ and with some appropriate right/left convexity assumptions, you can prove the maximum is attained at $(t,t,t,t,\dots , 0)$ etc. – dezdichado Nov 16 '24 at 20:30
  • 1
    @dezdichado Could you maybe put a full answer with this as an assumption? – Boby Nov 16 '24 at 20:36
  • 1
    My another idea is to prove (or disprove) this conjecture (I ran some numerical simulation which support it): Conjecture 1. Let $a > 0$ and $x > y > 0$ with $x + y \le 1$. Then $$x^{(1-x)^a} + y^{(1-y)^a}< \max\left((x+y)^{(1-x-y)^a}, , 2\left(\frac{x + y}{2}\right)^{(1 - (x+y)/2)^a}\right).$$ This idea is similar to my answer here: https://math.stackexchange.com/questions/722103/maximum-of-1-q-1-ldots1-q-n-for-non-negatives-sum-q-i2-1/4997424#4997424 – River Li Nov 20 '24 at 03:08

4 Answers4

6

Some thoughts.

Here we deal with the case $0 < a \le 2$. Let $f(x) := x^{(1-x)^a}$.

Fact 1. $f(x)$ is convex on $[0, c]$, and concave on $[c, 1]$, for some $c \in (0, 1)$.
(The proof is given at the end.)

Now we proceed. WLOG, assume that $p_1 \le p_2 \le \cdots \le p_n$. Let $$F(p_1, p_2, \cdots, p_n) := f(p_1) + f(p_2) + \cdots + f(p_n).$$ Then $F$ is maximal for $p_1 = \cdots = p_{k-1} = 0$ and $p_{k+1} = p_{k+2} = \cdots = p_n$ for some $k \in \{1, 2, \cdots, n\}$. The proof is easy. But many year ago, I knew Bin Zhao's half concave half convex theorem. So see the proof there.

(Edit 2024/12/03) Theorem 1. Let $a, b, c$ be fixed reals ($a < c < b$), and let $x_1, x_2, \cdots, x_n \in [a, b]$ ($n\ge 2$) with $x_1 + x_2 + \cdots + x_n = S$ (constant). Let $f$ be a continuous function on $[a, b]$, convex on $[a, c]$ and concave on $[c, b]$. Then $f(x_1) + f(x_2) + \cdots + f(x_n)$ is maximal for $x_1 = \cdots = x_{k-1} = a$ and $x_{k+1} = \cdots = x_n$ for some $k \in \{1, 2, \cdots, n\}$.

Proof of Theorem 1.

Let $Q$ be the set of all maximizers. Let $M(x_1, \cdots, x_n) := \#\{i : a < x_i < c, 1 \le i \le n\}$. Let $$p := \min_{(x_1, \cdots, x_n) \in Q} M(x_1, \cdots, x_n).$$

We claim that $p \le 1$. Assume, for the sake of contradiction, that $p \ge 2$. Let $(x_1, \cdots, x_n) \in Q$ with $M(x_1, \cdots, x_n) = p$. Then there exist $i < j$ such that $x_i, x_j \in (a, c)$.

(1) If $x_i + x_j - c \ge a$, by Karamata's inequality, we have $$f(x_i) + f(x_j) \le f(x_i + x_j - c) + f(c).$$ If we replace $x_i, x_j$ with $x_i + x_j - c, c$, we have a strictly smaller value of $M$.

(2) If $x_i + x_j - c < a$, by Karamata's inequality, we have $$f(x_i) + f(x_j) \le f(x_i + x_j - a) + f(a).$$ If we replace $x_i, x_j$ with $x_i + x_j - a, a$, we have a strictly smaller value of $M$.

For both cases, we get a contradiction. The claim is proved.

Now, WLOG, assume that $a \le x_1 \le x_2 \le \cdots \le x_n \le b$. Then there exists $k \in \{1, 2, \cdots, n\}$ such that $x_1 = \cdots = x_{k-1} = a$ and $c \le x_{k+1} \le \cdots \le x_n$. Also, we have $$f(x_{k+1}) + f(x_{k+2}) + \cdots + f(x_n) \le (n-k)f\left(\frac{x_{k+1} + \cdots + x_n}{n-k}\right)$$ with equality if $x_{k+1} = \cdots = x_n$. The desired result follows.

$\phantom{2}$


Cleary $k < n$ (when $k=n$, $(p_1, \cdots, p_n) = (0, \cdots, 0, 1)$ which is not a maximizer). Let $p_k = y$. Then $y_{k+1} = \cdots = y_n = \frac{1-y}{n-k}$. We have $$F = y^{(1-y)^a} + (n-k)\left(\frac{1-y}{n-k}\right)^{\left(1-\frac{1-y}{n-k}\right)^a}.$$

Can we proceed?

$\phantom{2}$


Proof of Fact 1.

We have $$f''(x) = x^{(1-x)^a}(1-x)^a g(x)$$ where \begin{align*} g(x) &:= \left(- \frac{a\ln x}{1-x} + \frac{1}{x}\right)^2(1-x)^a + \frac{a^2\ln x}{(1-x)^2} - \frac{a\ln x}{(1-x)^2} - \frac{2a}{x(1-x)} - \frac{1}{x^2}. \end{align*}

We can prove that $g'(x) < 0$ on $(0, 1)$ (the proof is given at the end).

Note that $\lim_{x\to 0^{+}} g(x) = \infty$ and $\lim_{x\to 1^{-}} g(x) = -\infty$. Thus, there exists $c \in (0, 1)$ such that $g(x) > 0$ on $(0, c)$ and $g(x) < 0 $ on $(c, 1)$. Thus, there exists $c \in (0, 1)$ such that $f''(x) > 0$ on $(0, c)$ and $f''(x) < 0 $ on $(c, 1)$. The desired result follows.

$\phantom{2}$


Proof of $g'(x) < 0$.

After some manipulations, it suffices to prove that, for all $0 < x < 1$, $$M (1-x)^a + N < 0,$$ where ($Q := \ln x$) \begin{align*} M &:= -(1 - x - Qax)[a(2-a)x^2Q + (1-x)(3ax-2x+2)],\\ N &:= 2\,{a}^{2}Q{x}^{3}-2\,aQ{x}^{3}-{a}^{2}{x}^{3}+{a}^{2}{x}^{2}+5\,a{x}^ {3}-7\,a{x}^{2}-2\,{x}^{3}\\ &\qquad +2\,ax+6\,{x}^{2}-6\,x+2. \end{align*} It is easy to prove that $M \le 0$.

If $0 < a \le 1$, by Bernoulli inequality, we have $(1-x)^a = \frac{1-x}{(1-x)^{1-a}} \ge \frac{1-x}{1 - x(1-a)}$, and it suffices to prove that $$M \cdot \frac{1-x}{1 - x(1-a)} + N < 0, $$ or $$m_2 a^2 + m_1 a + m_0 < 0,$$ where \begin{align*} m_2 &= {Q}^{2}{x}^{3}-{Q}^{2}{x}^{2}+2\,Q{x}^{3}-{x}^{3}+{x}^{2},\\ m_1 &= -2\,{Q}^{2}{x}^{3}+2\,{Q}^{2}{x}^{2}-6\,Q{x}^{2}+6\,{x}^{3}+4\,Qx-9\,{ x}^{2}+3\,x,\\ m_0 &= -2\,Q{x}^{3}+8\,Q{x}^{2}-4\,{x}^{3}-8\,Qx+9\,{x}^{2}+2\,Q-6\,x+1. \end{align*} We can prove that $m_2, m_1, m_0 < 0$ on $(0, 1)$. The desired result follows.

If $1 < a \le 2$, by Bernoulli inequality, we have $(1-x)^a = \frac{(1-x)^2}{(1-x)^{2-a}} \ge \frac{(1-x)^2}{1-x(2-a)}$, and it suffices to prove that $$M \cdot \frac{(1-x)^2}{1-x(2-a)} + N < 0,$$ or $$q_3a^3 + q_2a^2 + q_1 a + q_0 < 0, \tag{B1}$$ where \begin{align*} q_3 &= -{Q}^{2}{x}^{4}+2\,{Q}^{2}{x}^{3}-{Q}^{2}{x}^{2}+2\,Q{x}^{3}-{x}^{3}+{ x}^{2},\\ q_2 &= 2\,{Q}^{2}{x}^{4}-4\,{Q}^{2}{x}^{3}-4\,Q{x}^{4}+2\,{Q}^{2}{x}^{2}+6\,Q {x}^{3}-10\,Q{x}^{2}\\ &\qquad +7\,{x}^{3}+4\,Qx-10\,{x}^{2}+3\,x,\\ q_1 &= 4\,Q{x}^{4}-10\,Q{x}^{3}-3\,{x}^{4}+16\,Q{x}^{2}-10\,Qx+7\,{x}^{2}+2\, Q-5\,x+1,\\ q_0 &= 2\,{x}^{4}-6\,{x}^{3}+6\,{x}^{2}-2\,x. \end{align*} (B1) is true (the proof is smooth).

We are done.

River Li
  • 49,125
  • Thanks for your answer. I think I will accept it. One thing that I cannot follow as of yet. Is the proof of the theorem that you have linked. Specifically, I cannot understand this claim: case(ii) assume $m,(1 \le m \le i)$ be the minimum interger such that $x_1+x_2+\cdots+x_m-(m-1)a \ge c$ then we obtian $f(x_1)+f(x_2)+\cdots+f(x_m) \ge (m-1)f(a)+f(x_1+x_2+\cdots+x_{m-1}-(m-2)a)+f(x_m) \ge (m-1)f(a)+f(x_1+x_2+\cdots+x_{m-1}+x_m-c-(m-2)a)+f(c)$ Could you help me with it? – Boby Dec 03 '24 at 02:25
  • @Boby I will look at it. By the way, we can obtain a upper bound based on my result (as you mentioned in your question, upper bound is allowed). – River Li Dec 03 '24 at 02:31
  • I agree. But I think your answer is more or less complete at it leave to optimizer only over two parameters. Would appreciate your help with the theorem you cite. I can't quite follow all the steps. thanks – Boby Dec 03 '24 at 02:38
  • @Boby Did you see the linked page #18? There is a typo. – River Li Dec 03 '24 at 03:06
  • Thanks. Yeah, I saw that. I follow the first inequality but the second inequality I do not. Namely: (−2)()+(1+2+⋯+−1−(−2))+()≥(−2)()+(1+2+⋯+−1+−−(−2))+() Do you know what this step is? – Boby Dec 03 '24 at 03:59
  • I also a bit confused about induction. What does he mean that we have reduced it to $i-1$ case? – Boby Dec 03 '24 at 04:02
  • @Boby I wrote my proof of Theorem 1 (see the update). Please take a look. By the way, I think he used the Karamata's inequality. Some details are missed in his proof. – River Li Dec 03 '24 at 12:03
  • I see that you didn't use the induction argument anymore. – Boby Dec 03 '24 at 12:08
  • @Boby Yes. I just now notice that you asked the question yesterday. – River Li Dec 03 '24 at 12:26
4

Note: It appears my previous answer wasn't very rigorous and had gaps in its reasoning (especially with $n-1$ EV). I've edited this post with some more progress I've made on this problem.

Going off River Li's answer, we know the maximum for $0< a\leq 2$ occurs when $p_k=y$, $p_1=p_2=\dots=p_{k-1}=\frac{1-y}{k-1}\geq y$, and $p_{k+1}=p_{k+2}=\dots=p_n=0$. (I've slightly changed the indexing method for convenience). Now, this simplifies the problem into maximizing $$g(y)=y^{(1-y)^a}+(k-1)\left(\frac{1-y}{k-1}\right)^{\left(1-\frac{1-y}{k-1}\right)^a},$$ for $y\leq\frac 1k$. Then, we have the following claim:

Claim: If $g''(y)$ is convex on $(0,c)$ and concave on $(c,1)$ for some $c$, then the maximizing solution is in the form $p_1=\dots=p_k=\frac 1k$ and $p_{k+1}=\dots =p_n=0$.

Remark: The convex-concave property of $g''(y)$ appears to hold numerically, but I'm still working on a proof for this. Any ideas are appreciated.

Proof of Claim. We can write $g(y)=f(y)+(k-1)f\left(\frac{1-y}{k-1}\right)$ for $f(y)=y^{(1-y)^a}$. Now, note that $g'(y)=f'(y)-f'\left(\frac{1-y}{k-1}\right)$, which has a zero at $y=\frac{1}{k}$.

Now, assume $g''(y)=f''(y)+\frac{1}{k-1}f''\left(\frac{1-y}{k-1}\right)$, is convex on $(0,c)$ and concave on $(c,1)$ for some $c$. If $y=\frac 1k$ is the local minimum, then the maximum occurs at $y=0$ (recall $y\leq\frac 1k$). If $y=\frac 1k$ is the local maximum, then the maximum occurs at $y=0$ or $y=\frac 1k$.

In either case, it follows that the maximizing solution set consists of a certain number of equal values and the rest of the values being $0$.

Note: A full, rigorous solution of this problem now reduces to the following:

  1. Extending of River Li's answer to $a>2$.
  2. Showing $g''(y)$ satisfies the above convex-concave property.

Then, the maximizing solution set reduces to that of Dr. Wolfgang Hintze's post, where direct comparison can be used to find the maximizing set for every $a$.

masky
  • 1,594
  • (+1). I just look at your answer. I have a question about the (n-1)-EV in page 6 of the handout which refers to page 15 of Olympiad Inequalities, by Thomas Mildorf, do you think it is [this document] (https://artofproblemsolving.com/articles/files/MildorfInequalities.pdf)? (see next comment) – River Li Nov 20 '24 at 02:19
  • @RiverLi Yep, I think that's the document. The $n-1$ EV principle is proved for a specific function as a lemma in the proof of problem 21 (but the proof applies more generally for all functions with 1 inflection point). The footnote on page 15 states the principle explicitly. – masky Nov 20 '24 at 02:45
  • (continued) I have a question about applying the (n-1)-EV theorem. Let us see an example (by Ji Chen). Let $x, y, z \ge 0$ with $x + y + z = 5$. Let $f(x) = -5x^4 + 28x^3$. Then $f''(x) = -12x(5x - 14)$. So, $f(x)$ is convex on $[0, 14/5]$ and concave on $[14/5, 5]$ (has one inflection point). However, the maximum of $f(x) + f(y) + f(z)$ is attained when $(x, y, z) = (0, 1, 4)$, and the maximum does not occur when $n-1=2$ variables are equal to each other. Am I missing something? – River Li Nov 20 '24 at 02:50
  • @RiverLi This is because $x=0$ is technically an inflection point ($f''(x)=0$) if you include 0 in the bound you're considering $f$ on. So for $x,y,z\gneq 0$, n-1 EV principle applies. For $x,y,z\geq 0$, you would have to consider the edge case of one of $x,y,z$ being 0. – masky Nov 20 '24 at 03:13
  • But we can consider $x, y, z \ge 1/10$ and $x + y + z = 5$ (the maximum occurs when $(1/10, 0.846099, 4.0539)$), right? I think the point is that in the handout, it is for real numbers $x, y, z$, $f(x)$ has one inflection point on $(-\infty, \infty)$. See post 1 theorem 2, 2' in this link: https://artofproblemsolving.com/community/c6h64933 – River Li Nov 20 '24 at 03:17
  • @RiverLi You're right. I think the original $n-1$ EV stated that the function had to be from $\mathbb R\rightarrow\mathbb R$. However, I think the Karamata idea used to prove $n-1$ EV it can be extended to show that if the function has an inflection point on some bounded interval, then the maximum is either at $n-1$ equal values or on the bound. I haven't worked through the specifics of this, but if this is right, then the proof outline I've posted above should still hold. – masky Nov 20 '24 at 03:39
  • I think that we can not apply (n-1)-EV theorem in the handout for the OP. If $f(x)$ has exactly one inflection point on $[a, b]$, we can not say $(n-1)$ variables are equal to each other. We can say $x_1 = \cdots = x_{k-1} = a$ and $x_{k+1}= \cdots = x_n$ for some $k\in {1, \cdots, n}$ as stated in the link (also my answer). – River Li Nov 20 '24 at 03:53
  • @RiverLi You're right, sorry about that. It looks like that $n-1$ EV doesn't work :( – masky Nov 20 '24 at 04:29
  • Thanks for discussion! I guess that the solution is something like what you described though we can not use n-1 EV. – River Li Nov 20 '24 at 05:21
  • Your claim is nice. – River Li Nov 26 '24 at 07:16
4

Update Nov. 23, 2024

I have continued my experiments and I believe I have found a method to calculate the "phase"-transition points for the parameter $a$, and have - in view of the seeming complexity of the problem suprisingly - obtained an "almost" analytic solution.

The solution

The solution can be given in terms of a simple function the domain of which is determined by a set of numerically determined global transition constants.

Here I present just the solution leaving explanations for later.

Recall that defining the sum

$$s(a,n,\{p\}) = \sum_{i=1}^{n} p_{i}^{(1-p_{i})^a} \tag{1}$$

where $a>0$ is a real parameter and $p_{i}\in [0,1]$ are probabilities subject to $\sum_{i=1}^{n} p_{i}=1 $ we are looking, for given $n$ and $a$, for the maximum $s_{max}$ of $s$ over all possible sets $\{p\}=\{p_{1}, p_{2}, \cdots , p_{n}\}$.

Now for $k\ge 2$ define the set of functions

$$f(k,a)=k^{1-\left(1-\frac{1}{k}\right)^a}\tag{2}$$

then the solution is given by

$$s_{max}(n,a)=max(f(2,a), f(3,a), \cdots, f(n,a))\\ = \max \left(2^{1-\left(\frac{1}{2}\right)^{a}},3^{1-\left(\frac{2}{3}\right)^a},\cdots,n^{1-\left(\frac{n-1}{n}\right)^a}\right) \tag{3}$$

This can be made more explicit by calculating first the set of transition points $a_t(k)$ between each two adjacent functions $f(k,a)$ defined by the solution of the equation

$$f(k,a_t(k))=f(k+1, a_t(k))\tag{4}$$

This equation can be solved iteratively starting with the inial value $a_t(k) = 2k$.

The first 10 values, starting at $k=2$, are

$$a_{t}(k=2..10)=\{0.553953,2.13996,3.87677,5.72384,\\ 7.65616,9.65727,11.7156,13.8228,15.9724\}\tag{5}$$

Then we can write $s_{max}$ as this piecewise function

$$s_{max}(n,a) = \text{IF } a\in (a_t(k),a_t(k+1)) \text{ THEN} f(k,a) \tag{6}$$

Note that $s_{max}(n,a)$ is a continuous function of $a$ but it is not smooth, i.e. its derivative is not continuous but has (finite) jumps at the transition points.

Example solutions

The general behaviour of the solution can be described as follows, also illustrated by the simulations:

For $n \gt2$ and very small $a$ the maximum is $a_{max}(n,a)=f(2,a)$ corresponding to the probability set $\{p\} = \{\frac{1}{2},\frac{1}{2}, 0,\cdots,0\}$, when a grows beyond the transition point $a_t(2)$ the maximum becomes $a_{max}(n,a)=f(3,a)$ corresponding to the set $\{p\} = \{\frac{1}{3},\frac{1}{3}, \frac{1}{3}, 0, \cdots,0\}$, and so on up to $a_t(n-1)<a<a_t(n)$ where $a_{max}(n,a)=f(n-1,a)$ with $\{p\} = \{\frac{1}{n-1},\cdots,\frac{1}{n-1},0\}$ and, finally, for $a>a_t(n)$ where $a_{max}(n,a)=f(n,a)$ with $\{p\} = \{\frac{1}{n},\cdots,\frac{1}{n}\}$.

(Examples and graphs $s_{max}(a)$ to be done)

Original post

In order to get an overwiew of the solutions I have carried out some Monte-Carlo simulations and found an interesting phase transition like behaviour.

Let us rephrase the problem. Define the sum

$$s(a,n,\{p\}) = \sum_{i=1}^{n} p_{i}^{(1-p_{i})^a} \tag{1}$$

where $a>0$ is a real parameter and $p_{i}\in [0,1]$ are probabilities subject to $\sum_{i=1}^{n} p_{i}=1 $.

For given $n$ and $a$ we are looking for the maximum $s_{max}$ of $s$ over all possible sets $\{p\}=\{p_{1}, p_{2}, \cdots , p_{n}\}$.

Monte Carlo pseudo code

In order to find the maximum for given parameter $a$ we set up a Monte-Carlo simulation in the following loop

10 max = 0
20 get a set of $n$ random probabilities $\{p\}$ as defined above
30 calculate s
40 If s>max then (max = s; $\{p\}_{max} = \{p\}$)
50 goto 20

The loop is carried out a certain number $r$ of times until the maximum approaches a constant value.

Notice that we obtain not only the value of the maxiumum but - and this is even more interesting - the set of probabilities leading to that maximum.

Results

Results of a preliminary study are

For given $n= 2 .. 7$ and for $a=\{0.1, 1, 10\}$ we calculate the maximum of $s$ ad depict the set of probabilities corresponding to that value.

Actually, for the time beeing I found it more interesting to study the structure of the peobability sets than the maximum (which I shall provide soon).

All values (except for n=2) of the probabilities are approximated by simple rationals in order to exhibit the structure of the set more clearly.

We can see that, with varying $a$, there are interesting transistions in the structure of the probability sets. These remind (me) of phase transistions.

This should be studies in more depth but the available results already seem to be interesting.

$n=2$ ($r = 10^5$)
$a=0.1$ -> $\{p\} =$ {$\frac {1}{2}$,$\frac {1}{2}$}
$a=1.0$ -> $\{p\} =$ {$\frac {1}{2}$,$\frac {1}{2}$}
$a=10$ -> $\{p\} =$ {$\frac {1}{2}$,$\frac {1}{2}$}

$n=3$ ($r = 10^5$)
$a=0.1$ -> $\{p\} =$ {$\frac {1}{2}$,$\frac {1}{2}$,$0$}
$a=0.55$ -> $\{p\} =$ {$\frac {1}{2}$,$\frac {1}{2}$,$0$}
$a=0.56$ -> $\{p\} =$ {$\frac {1}{3}$,$\frac {1}{3}$,$\frac {1}{3}$}
$a=1.0$ -> $\{p\} =$ {$\frac {1}{3}$,$\frac {1}{3}$,$\frac {1}{3}$}
$a=10$ -> $\{p\} =$ {$\frac {1}{3}$,$\frac {1}{3}$,$\frac {1}{3}$}

$n=4$ ($r = 10^5$)
$a=0.1$ -> $\{p\} \simeq$ {$\frac {1}{2}$,$\frac {1}{2}$,$0$,$0$}
$a=0.55$ -> $\{p\} =$ {$\frac {1}{2}$,$\frac {1}{2}$,$0$,$0$}
$a=0.56$ -> $\{p\} =$ {$\frac {1}{3}$,$\frac {1}{3}$,$\frac {1}{3}$,$0$}
$a=1.0$ -> $\{p\} =$ {$\frac {1}{3}$,$\frac {1}{3}$,$\frac {1}{3}$,$0$}
$a=2.1$ -> $\{p\} =$ {$\frac {1}{3}$,$\frac {1}{3}$,$\frac {1}{3}$,$0$}
$a=2.2$ -> $\{p\} =$ {$\frac {1}{4}$,$\frac {1}{4}$,$\frac {1}{4}$,$\frac {1}{4}$}
$a=10$ -> $\{p\} =$ {$\frac {1}{4}$,$\frac {1}{4}$,$\frac {1}{4}$,$\frac {1}{4}$}

$n = 5$ ($r = 10^5$)
$a = 0.1$ -> $\{p\} =$ {$\frac {1}{2}$,$\frac {1}{2}$,$0$,$0$,$0$}
$a = 0.47$ -> $\{p\} =$ {$\frac {1}{2}$,$\frac {1}{2}$,$0$,$0$,$0$}
$a = 0.48$ -> $\{p\} =$ {$\frac {1}{3}$,$\frac {1}{3}$,$\frac {1}{3}$,$0$,$0$}
$a = 1.0$ -> $\{p\} =$ {$\frac {1}{3}$,$\frac {1}{3}$,$\frac {1}{3}$,$0$,$0$}
$a = 3.7$ -> $\{p\} =$ {$\frac {1}{3}$,$\frac {1}{3}$,$\frac {1}{3}$,$0$,$0$}
$a = 3.8$ -> $\{p\} =$ = {$\frac {1}{5}$,$\frac {1}{5}$,$\frac {1}{5}$,$\frac {1}{5}$,$\frac {1}{5}$}
$a = 10$ -> $\{p\} =$ = {$\frac {1}{5}$,$\frac {1}{5}$,$\frac {1}{5}$,$\frac {1}{5}$,$\frac {1}{5}$}

$n = 6$ ($r = 10^5$)
$a = 0.1$ -> $\{p\} =$ {$\frac {1}{2}$,$\frac {1}{2}$,$0$,$0$,$0$,$0$}
$a = 1.0$ -> $\{p\} =$ {$\frac {1}{3}$,$\frac {1}{3}$,$\frac {1}{3}$,$0$,$0$,$0$}
$a = 10$ -> $\{p\} =$ {$\frac {1}{6}$,$\frac {1}{6}$,$\frac {1}{6}$,$\frac {1}{6}$,$\frac {1}{6}$,$\frac {1}{6}$}}

$n = 7$ ($r = 10^5$)
$a = 0.1$ -> $\{p\} =$ {$\frac {1}{2}$,$\frac {1}{2}$,$0$,$0$,$0$,$0$,$0$}
$a = 1.0$ -> $\{p\} =$ {$\frac {1}{3}$,$\frac {1}{3}$,$\frac {1}{3}$,$0$,$0$,$0$,$0$}
$a = 10$ -> $\{p\} =$ {$\frac {1}{7}$,$\frac {1}{7}$,$\frac {1}{7}$,$\frac {1}{7}$,$\frac {1}{7}$,$\frac {1}{7}$,$\frac {1}{7}$}

Dr. Wolfgang Hintze
  • 13,265
  • 25
  • 49
3

Some thoughts 2.

In my previous answer, we dealt with the case $0 < a \le 2$. Here we deal with the case $a > 2$.

Fact 1. $f(x)$ is convex on $[0, c]$, and concave on $[c, 1]$, for some $c \in (0, 1)$.
(The proof is given at the end.)

Now we proceed. WLOG, assume that $p_1 \le p_2 \le \cdots \le p_n$. Let $$F(p_1, p_2, \cdots, p_n) := f(p_1) + f(p_2) + \cdots + f(p_n).$$ Then $F$ is maximal for $p_1 = \cdots = p_{k-1} = 0$ and $p_{k+1} = p_{k+2} = \cdots = p_n$ for some $k \in \{1, 2, \cdots, n\}$. The proof is easy. But many year ago, I knew Bin Zhao's half concave half convex theorem: https://artofproblemsolving.com/community/c6h64933. So see the proof there.

Cleary $k < n$ (when $k=n$, $(p_1, \cdots, p_n) = (0, \cdots, 0, 1)$ which is not a maximizer). Let $p_k = y$. Then $y_{k+1} = \cdots = y_n = \frac{1-y}{n-k}$. We have $$F = y^{(1-y)^a} + (n-k)\left(\frac{1-y}{n-k}\right)^{\left(1-\frac{1-y}{n-k}\right)^a}.$$

Can we proceed?

$\phantom{2}$


Proof of Fact 1.

We have $$f''(x) = x^{(1-x)^a}(1-x)^{2a} g(x)$$ where \begin{align*} g(x) &:= \left(- \frac{a\ln x}{1-x} + \frac{1}{x}\right)^2 + \left(\frac{a^2\ln x}{(1-x)^2} - \frac{a\ln x}{(1-x)^2} - \frac{2a}{x(1-x)} - \frac{1}{x^2}\right)\frac{1}{(1-x)^a}. \end{align*}

We can prove that $g'(x) < 0$ on $(0, 1)$ (the proof is given at the end).

Note that $\lim_{x\to 0^{+}} g(x) = \infty$ and $\lim_{x\to 1^{-}} g(x) = -\infty$. Thus, there exists $c \in (0, 1)$ such that $g(x) > 0$ on $(0, c)$ and $g(x) < 0 $ on $(c, 1)$. Thus, there exists $c \in (0, 1)$ such that $f''(x) > 0$ on $(0, c)$ and $f''(x) < 0 $ on $(c, 1)$. The desired result follows.

$\phantom{2}$


Proof of $g'(x) < 0$ on $(0, 1)$.

After some manipulations, it suffices to prove that, for all $0 < x < 1$, $$M (1 - x)^a + N < 0,$$ where ($Q = \ln x$) \begin{align*} M &:= - 2(-Qax - x + 1)[ax(Qx - x + 1) + (x-1)^2], \\ N &:= a \left( a+2 \right) \left( a-1 \right)x^3 Q \\ &\qquad +{a}^{2}{x}^{3}-{a}^{ 2}{x}^{2}+4\,a{x}^{3}-5\,a{x}^{2}-2\,{x}^{3}+ax+6\,{x}^{2}-6\,x+2. \end{align*} It is easy to prove that $M < 0$ on $(0, 1)$.

We claim that $N < 0$ for all $\frac{1}{a} \le x < 1$. Using $\ln x \le \frac{2(x-1)}{1+x}$ for all $x\in (0, 1)$, it suffices to prove that, for all $x\in [1/a, 1)$, \begin{align*} &a \left( a+2 \right) \left( a-1 \right)x^3 \cdot \frac{2(x - 1)}{1 + x} \\ &\qquad +{a}^{2}{x}^{3}-{a}^{ 2}{x}^{2}+4\,a{x}^{3}-5\,a{x}^{2}-2\,{x}^{3}+ax+6\,{x}^{2}-6\,x+2 < 0 \end{align*} which is true. The claim is proved

Thus, it suffices to prove that, for all $0 < x < 1/a$, $$M(1 - x)^a + N < 0. $$

Using Bernoulli inequality $(1+u)^r \ge 1 + ur$ for all $r \ge 1$ and $u \ge -1$, we have $(1-x)^a = (1-x)(1-x)^{a-1} \ge (1-x)[1 - x(a-1)]$. It suffices to prove that, for all $0 < x < 1/a$, $$M (1-x)[1 - x(a-1)] + N < 0, $$ or (after clearing the denominators) $$m_2 \ln^2 x + m_1 \ln x + m_0 < 0, \tag{1}$$ where \begin{align*} m_2 &:= 2a^2(1-x)x^2(1+x-ax), \\ m_1 &:= -2\,{a}^{3}{x}^{4}+4\,{a}^{3}{x}^{3}+6\,{a}^{2}{x}^{4}-{a}^{3}{x}^{2}- 12\,{a}^{2}{x}^{3}\\ &\qquad -4\,a{x}^{4}+7\,{a}^{2}{x}^{2}+6\,a{x}^{3}-6\,ax+2\, a,\\ m_0 &:= -2\,{a}^{2}{x}^{4}+6\,{a}^{2}{x}^{3}+4\,a{x}^{4}-5\,{a}^{2}{x}^{2}-12 \,a{x}^{3}\\ &\qquad -2\,{x}^{4}+{a}^{2}x+16\,a{x}^{2}+6\,{x}^{3}-9\,ax-6\,{x}^{2 }+a+2\,x. \end{align*} Clearly, we have $m_2 > 0$.

Using $-\ln x \le \frac{1 - x^2}{2x}$ for all $0 < x < 1$, to prove (1), it suffices to prove that $$m_2 (-\ln x) \cdot \frac{1 - x^2}{2x} + m_1 \ln x + m_0 < 0, $$ or $$\left(- m_2 \cdot \frac{1 - x^2}{2x} + m_1 \right)\ln x + m_0 < 0. \tag{2}$$ We can prove that $- m_2 \cdot \frac{1 - x^2}{2x} + m_1 > 0$.

Using $\ln x \le \frac{2(x-1)}{1+x}$ for all $0 < x < 1$, to prove (2), it suffices to prove that $$\left(- m_2 \cdot \frac{1 - x^2}{2x} + m_1 \right)\cdot \frac{2(x-1)}{1+x} + m_0 < 0,$$ or \begin{align*} &2\,{a}^{3}{x}^{5}-6\,{a}^{3}{x}^{4}-2\,{a}^{2}{x}^{5}+6\,{a}^{3}{x}^{3 }+10\,{a}^{2}{x}^{4}-18\,{a}^{2}{x}^{3}\\ &\qquad -4\,a{x}^{4}+17\,{a}^{2}{x}^{2} +8\,a{x}^{3}-2\,{x}^{4}-3\,{a}^{2}x+2\,{x}^{3}\\ &\qquad -5\,ax+2\,{x}^{2}+3\,a-2 \,x > 0 \end{align*} which is true. We can prove it by the Buffalo Way (BW), i.e., use the substitution $a = 2 + t, x = \frac{1}{a} \cdot \frac{1}{1+s}$ for $s, t > 0$.

We are done.

River Li
  • 49,125